A question on Cantor's second diagonalization argument

In summary, Cantor used two diagonalization arguments in his proof. The first showed that the cardinality of the set of natural numbers is equal to the cardinality of the set of rational numbers. The second argument showed that the cardinality of the set of rational numbers is less than the cardinality of the set of real numbers. This was done by constructing a real number between 0 and 1 that cannot be put in a one-to-one correspondence with any natural number. Although there are some technicalities with the decimal expansions, the overall proof still holds.
  • #1
Organic
1,224
0
Hi,

Cantor used 2 diagonalization arguments.

On the first argument he showed that |N|=|Q|.

On the second argument he showed that |Q|<|R|.

I have some question on the second argument.

From Wikipedia, the free encyclopedia:
http://en.wikipedia.org/wiki/Cantor's_diagonal_argument

The proof by contradiction proceeds as follows:
• (1) Assume that the interval (0,1) is countably infinite.
• (2) We may then enumerate the numbers in this interval as a sequence, { r1, r2, r3, ... }
• (3) We shall now construct a real number x between 0 and 1 by considering the nth digit after the decimal point of the decimal expansion of rn. Assume, for example, that the decimal expansions of the beginning of the sequence are as follows.
r1 = 0 . 0 1 0 5 1 1 0 ...
r2 = 0 . 4 1 3 2 0 4 3 ...
r3 = 0 . 8 2 4 5 0 2 6 ...
r4 = 0 . 2 3 3 0 1 2 6 ...
r5 = 0 . 4 1 0 7 2 4 6 ...
r6 = 0 . 9 9 3 7 8 3 8 ...
r7 = 0 . 0 1 0 5 1 3 0 ...
...
The digits we will consider are indicated in bold. From these digits we define the digits of x as follows.

o if the nth digit of rn is 0 then the nth digit of x is 1
o if the nth digit of rn is not 0 then the nth digit of x is 0
For the example above this will result in the following decimal expansion.
x = 0 . 1 0 0 1 0 0 1 ...
The number x is clearly a real number between 0 and 1.
• (4) However, it differs in the nth decimal place from rn, so x is not in the set { r1, r2, r3, ... }.
• (5) This set is therefore not an enumeration of all the reals in the interval (0,1).
• (6) This contradicts with (2), so the assumption (1) that the interval (0,1) is countably infinite must be false.

Note: Strictly speaking, this argument only shows that the number of decimal expansions of real numbers between 0 and 1 is not countably infinite. But since there are expansions such as 0.01999... and 0.02000... that represent the same real number, this does not immediately imply that the corresponding set of real numbers is also not countably infinite. This can be remedied by disallowing the decimal expansions that end with an infinite series of 9's. In that case it can be shown that for every real number there is a unique corresponding decimal expansion. It is easy to see that the proof then still works because the number x contains only 1's and 0's in its decimal expansion.


My question is:

The first assumption was that we have a bijection between |N| and |R| iff R list is complete.

Then Cantor showed that there is a way to construct a real number x between 0 and 1, that cannot be put in 1-1 correspondence with any natural number.

Therefore, there are more real numbers between 0 and 1 then all natural numbers.

Now I construct the list in another way.

...
r7' = 0 . 0 1 0 5 1 3 8 ...
r6' = 0 . 9 9 3 7 8 0 8 ...
r5' = 0 . 4 1 0 7 0 4 6 ...
r4' = 0 . 2 3 3 5 1 2 6 ... New list
r3' = 0 . 8 2 0 5 0 2 6 ...
r2' = 0 . 4 0 3 2 0 4 3 ...
r1' = 0 . 1 0 0 1 0 0 1 ...(= Cantor's switching function resultes)
----------------------
r1 = 0 . 0 1 0 5 1 1 0 ...
r2 = 0 . 4 1 3 2 0 4 3 ...
r3 = 0 . 8 2 4 5 0 2 6 ...
r4 = 0 . 2 3 3 0 1 2 6 ... Original list
r5 = 0 . 4 1 0 7 2 4 6 ...
r6 = 0 . 9 9 3 7 8 3 8 ...
r7 = 0 . 0 1 0 5 1 3 0 ...
...

By constructing the above list I show that there are infinitely countably many r' numbers that are not covered by Cantor's switching function.

Therefore Cantor's digitalization's second argument doesn't hold.


Please someone show my mistake.


Thank you.


Organic
 
Mathematics news on Phys.org
  • #2
Uh, that you didn't understand Cantor's argument?

In the first, place, it is not an "assumption" that "we have a bijection between |N| and |R| iff R list is complete." Since N is countable, any bijection must be a complete list. It is incorrect to say (as you may be) that Cantor is saying that there is no bijection because that particular list is not complete- the list was arbitrary. What Cantor showed was that ANY list must be incomplete.

By constructing the above list I show that there are infinitely countably many r' numbers that are not covered by Cantor's switching function.
What do you mean "not covered by Cantor's switching function"?

You appear to have constructed the same list except that you have replace the "diagonal" digit by something that is neither the original digit nor Cantor's new digit. What is your point? There is no reason to believe you HAVE a "new" list. It is quite possible that the numbers you have in the new list were already somewhere in the old list- you don't know. An important point about going specifically down the "diagonal" in Cantor's proof is showing the the resulting new number CAN'T be in the list anywhere- because it differs from the nth number in the nth digit.

In any case, Cantor did NOT say "if the number is not 1, replace it by 1, if it is 1 replace it by 0". He only needs to set up some regular scheme for replacing each digit by some other digit in a clear way. In fact, what that shows is that, given any list, not only does there exist A number not on the list, but in fact, there exist an infinite number of real numbers not on the list.
 
  • #3
Hi HallsofIvy,


By complete I mean that there are no numbers left out not in R and not in N (means bijection between N and R).

The R list is arbitrary and it is one and only one list.
"New list" and "Original list" are the same one list.

All what I showed is that we can construct it in such a way that Cantor's function can never reach all of it.

Here it is again:
...
r7' = 0 . 0 1 0 5 1 3 8 ...
r6' = 0 . 9 9 3 7 8 0 8 ...
r5' = 0 . 4 1 0 7 0 4 6 ...
r4' = 0 . 2 3 3 5 1 2 6 ... "New list"
r3' = 0 . 8 2 0 5 0 2 6 ...
r2' = 0 . 4 0 3 2 0 4 3 ...
r1' = 0 . 1 0 0 1 0 0 1 ...(= Cantor's function resultes)
----------------------
r1 = 0 . 0 1 0 5 1 1 0 ...
r2 = 0 . 4 1 3 2 0 4 3 ...
r3 = 0 . 8 2 4 5 0 2 6 ...
r4 = 0 . 2 3 3 0 1 2 6 ... "Original list"
r5 = 0 . 4 1 0 7 2 4 6 ...
r6 = 0 . 9 9 3 7 8 3 8 ...
r7 = 0 . 0 1 0 5 1 3 0 ...
...

You Wrote:
Let's write down an enumeration of your list:

r1 = 0.0105110...
r1' = 0.1001001...
r2 = 0.4132043...
r2' = 0.4032043...
r3 = 0.8245026...
r3' = 0.8205026...
...

Then I use r'', r''' ,r'''' ,... on top of your list.


Organic
 
Last edited:
  • #4
Then I use r'', r''' ,r'''' ,... on top of your list.

And the diagonal argument applies to each of those as well.
 
  • #5
Hi Hyrkyl,

As I see I it, we Don't need r'' ,r''' , r'''' ,... and so on.

r' is enough to show that Coator's argument can't work, if we construct the list as I did.

There are always countably many numbers that Cantor's function does not reach.

More than that, because it is an arbitrery list, there is no first number in the list, and this is how I constructed it.

It always goes to infinity in both directions but only a part of it is in the the range of Contor's function:
...
r7' = 0 . 0 1 0 5 1 3 8 ...
r6' = 0 . 9 9 3 7 8 0 8 ...
r5' = 0 . 4 1 0 7 0 4 6 ...
r4' = 0 . 2 3 3 5 1 2 6 ... "Out of Contor's function range"
r3' = 0 . 8 2 0 5 0 2 6 ...
r2' = 0 . 4 0 3 2 0 4 3 ...
r1' = 0 . 1 0 0 1 0 0 1 ...(= Cantor's function resultes)
r1 = 0 . 0 1 0 5 1 1 0 ...
r2 = 0 . 4 1 3 2 0 4 3 ...
r3 = 0 . 8 2 4 5 0 2 6 ...
r4 = 0 . 2 3 3 0 1 2 6 ... "In range"
r5 = 0 . 4 1 0 7 2 4 6 ...
r6 = 0 . 9 9 3 7 8 3 8 ...
r7 = 0 . 0 1 0 5 1 3 0 ...
...



Please show me what mathematical logic forces me to define the first row to the checked list ?




(Dear Hurkyl please close or delete the other thread with the wrong name "digitalization" instead of "diagonalization", thank you)

Organic
 
Last edited:
  • #6
See step (2) in the proof you wrote in the original post:

• (2) We may then enumerate the numbers in this interval as a sequence, { r1, r2, r3, ... }


This is a key step to the diagonal argument that you are neglecting.

You have a (countable) list, r' of decimals in the interval (0, 1). Your list may be enumerated as a sequence {s1, s2, s3, ...}, and the sequence s has exactly the same elements as r' does. Steps (3)-(5) prove the existence of a decimal, x, in (0, 1) that is not in the enumeration s, thus x must also not be in r'.

(a previous post gives an example of such a sequence s)


No matter how much effort you make in creating (countable) lists structured in complicated ways trying to circumvent this proof, step (2) always allows the list to be rewritten as a sequence and the proof holds.
 
  • #7
In other words, you don't understand the word "countable", you don't understand the word "list", and you don't understand Cantor's proof.
 
  • #8
Thank you Hurkyl and HallsofIvy,

Cantor's proof holds because one and only one reason.

It compares between a list of numbers that have finite number of digits or infinitely many digits (with repetitions over scale, therefore they are Q numbers), and a list of numbers with infinitely many digits without repetitions over scales (irrational numbers).

If we take a list of all Q numbers with infinitely many digits and use Cantor's function, can we clearly show that we always get as a result an irrational number ?

Thank you.


Organic
 
Last edited:
  • #9
Cantor's proof holds because one and only one reason.

It compares between a list of numbers that have finite number of digits or infinitely many digits (with repetitions over scale, therefore they are Q numbers), and a list of numbers with infinitely many digits without repetitions over scales (irrational numbers).


The (second) diagonalization argument has nothing to do with rational and irrational numbers.


If we take a list of all Q numbers with infinitely many digits and use Cantor's function, can we clearly show that we always get as a result an irrational number ?

If we take a list of all rational numbers and we use the diagonal argument to prove the existence of a real number not on that list, then of course that number has to be irrational.
 
  • #10
Hi Hurkyl,

Thank you for your reply.

My question is: can we prove that the new number can never be a rational number ?

I mean no repetitions over scales.


I can construct a list of infinitely many rational numbers and than I can show that there is a new rational numbet, which is not in the list.

For example:

0 . 1 7 1 1 3 1 7 1 1 3 1 7 ...
0 . 3 0 2 3 0 2 3 0 2 3 0 2 ...
0 . 4 2 1 3 4 2 1 3 4 2 1 3 ...
0 . 5 5 9 0 6 5 5 9 0 6 5 5 ...
0 . 7 8 1 1 1 7 8 1 1 1 7 8 ...
0 . 7 4 3 3 3 0 7 4 3 3 3 0 ...
0 . 3 5 4 9 5 5 1 3 5 4 9 5 ...
0 . 1 2 3 0 1 2 3 0 1 2 3 0 ...
0 . 6 4 1 6 4 1 6 4 1 6 4 1 ...
0 . 3 0 2 0 3 0 2 0 3 0 2 0 ...
0 . 6 1 3 6 1 3 6 1 3 6 1 3 ...
0 . 2 7 1 0 2 7 1 0 2 7 1 0 ...
...
...
...

In this case x = 0.010101010101... , which is a new rational number not in the list of infinitely many rational numbers.

Therefore (by Cantor's first and second agruments) there are more rational numbers than rational numbers.

If we add this number to the list, we can rearenge the list in such a way that define a new rational number not in the list, and so on and so on.


Can you prove that the number of rearengments is finite ?



Thank you.




Organic
 
Last edited:
  • #11
Therefore (by Cantor's first and second agruments) there are more rational numbers than rational numbers.

How does that follow?

Are you assuming that the list you provided contains every rational number? Why would you think that?

Cantor's first diagonal argument constructs a specific list of the rational numbers that is not the list you provided.
 
  • #12
Hi Hurkyl,

My list is a decimal representation of any rational number in Cantor's first argument spesific list.

For example:

0 . 1 7 1 1 3 1 7 1 1 3 1 7 ...
1 . 0 0 0 0 0 0 0 0 0 0 0 0 ...
0 . 4 2 1 3 4 2 1 3 4 2 1 3 ...
0 . 5 5 9 0 6 5 5 9 0 6 5 5 ...
0 . 7 8 1 1 1 7 8 1 1 1 7 8 ...
2 . 0 0 0 0 0 0 0 0 0 0 0 0 ...
0 . 3 5 4 9 5 5 1 3 5 4 9 5 ...
3 . 0 0 0 0 0 0 0 0 0 0 0 0 ...
0 . 6 4 1 6 4 1 6 4 1 6 4 1 ...
0 . 3 0 2 0 3 0 2 0 3 0 2 0 ...
0 . 6 1 3 6 1 3 6 1 3 6 1 3 ...
0 . 2 7 1 0 2 7 1 0 2 7 1 0 ...
...
...
...

Every non-zero decimal digit can be any number between 1 to 9.



Organic
 
Last edited:
  • #13
In this case x = 0.010101010101... , which is a new rational number not in the list of infinitely many rational numbers.

Therefore (by Cantor's first and second agruments) there are more rational numbers than rational numbers.
No, it proves that there are more rational numbers than were on your list. I can play that game too: 1, 2, 3, 4, ..., the list of all natural numbers, is a list of rational numbers. The number 1/2 is not on it! That proves nothing at all.

The point of Cantor's argument was to say "suppose we have a list of all real numbers" and then show that that leads to a contradiction.

If you are going to try to do the same thing with rational numbers- assume a list of all rational numbers and then construct a rational number that is not on it, the onus is on you to show that that number IS rational.

My list is a decimal representation of any rational number in Cantor's first argument spesific list.

This makes no sense at all. You haven't given a list, you give a few numbers, (not well defined, in fact it is not even clear that the numbers on the list ARE rational numbers) and no general formula for putting numbers on the list.
 
  • #14
My list is a decimal representation of any rational number in Cantor's first argument spesific list.

Really?

Where does 1/3 appear in your list? What about 4/7? How about 10/99?

(If you only answer one of these questions, do it for 10/99)
 
  • #15
Hi Hurkyl ,Hi HallsofIvy,


The main idea of Cnator's second argument is to show that the real numbers list can never be a complete list.

If we have to correct the list by adding to it infinitely many Cantor's function results, it means that there can never be a bijection between |R| and |N|.

Cantor's first argument clearly shows that there is a bijection between |N| and |Q|.

There is no problem to represent any rational number by its decimal form.

And Q numbers decimal's form is finite or it is infinitely many digits with repetitions over scales.

But when we use Cantor's second argument on the decimal representations of Q numbers, we find exactly the same results as Cantor found between |N| and |R|.

|N|=|Q| by Cantor's first argument, but |N|<|Q| by Cantor's second argument.

My list is a decimal representation of any rational number in Cantor's first argument specific list.

For example:

0 . 1 7 1 1 3 1 7 1 1 3 1 7 ...
1 . 0 0 0 0 0 0 0 0 0 0 0 0 ...
0 . 4 2 1 3 4 2 1 3 4 2 1 3 ...
0 . 5 5 9 0 6 5 5 9 0 6 5 5 ...
0 . 3 3 3 3 3 3 3 3 3 3 3 3 ...
2 . 0 0 0 0 0 0 0 0 0 0 0 0 ...
0 . 3 5 4 9 5 5 1 3 5 4 9 5 ...
3 . 0 0 0 0 0 0 0 0 0 0 0 0 ...
0 . 6 4 1 6 4 1 6 4 1 6 4 1 ...
0 . 3 0 2 0 3 0 2 0 3 0 2 0 ...
0 . 6 1 3 6 1 3 6 1 3 6 1 3 ...
0 . 2 7 1 0 2 7 1 0 2 7 1 0 ...
...
...
...

Hurkyl, every non-zero decimal digit can be any number between 1 to 9, Because I use Cantor's function where the rules are:

A) Every 0 in the original diagonal number is turned to 1 in Cantor's new number.

B) Every non-zero in the original diagonal number is turned to 0 in Cantor's new number.

So, there is no problem to use 0.333333... in the list because 3 (or any digit between 1 to 9) is always turned to 0, therefore our new number can always be a rational number.


Organic
 
Last edited:
  • #16
You miss understand the argument, what Cantor showed is that given any list of any size natural numbers vs irrational numbers between 1 and 0 it is always possible to prove that there are irrational numebsr not contained on the list, the reverse is not true however.
 
  • #17
Hi jcsd,

please go one post back form your first post in this thread, and please answer to my agrument about |N| and |Q|.

Thank you.


Organic
 
  • #18
If we have to correct the list by adding to it infinitely many Cantor's function results, it means that there can never be a bijection between |R| and |N|.

There is no bijection between |N| and |R| because every function from |N| to |R| is missing at least one real number.


Your use of the phrase "can never" seems to imply you think that, in the mathematical sense, something might not exist now but it could exist in the future. That is an incorrect interpretation of mathematics. A mathematical statement that is true now will be true 100 years from now and was true 1000 years ago; all that changes is what we've discovered.


So, there is no problem to use 0.333333... in the list because 3 (or any digit between 1 to 9) is always turned to 0, therefore our new number can always be a rational number.

So tell me, where does 10/99 appear in your list?
 
  • #19
Hi Hurkyl,

you wrote:
Your use of the phrase "can never" seems to imply you think that, in the mathematical sense, something might not exist now but it could exist in the future. That is an incorrect interpretation of mathematics. A mathematical statement that is true now will be true 100 years from now and was true 1000 years ago; all that changes is what we've discovered.
Well, I think it is depended on your point of view, which in this case is Platonism, which says that mathematics only discover timeless objective truth.

Let us say that I take this point of view, so I can easily change what I wrote to fit it to the Platonism point of view.

Here it is (and also added 10/99 to my list):

The main idea of Cnator's second argument is to show that the real numbers list can never be a complete list.

If we have to correct the list by adding to it infinitely many Cantor's function results, it means that there is no bijection between |R| and |N|.

Cantor's first argument clearly shows that there is a bijection between |N| and |Q|.

There is no problem to represent any rational number by its decimal form.

And Q numbers decimal's form is finite or it is infinitely many digits with repetitions over scales.

But when we use Cantor's second argument on the decimal representations of Q numbers, we find exactly the same results as Cantor found between |N| and |R|.

|N|=|Q| by Cantor's first argument, but |N|<|Q| by Cantor's second argument.

My list is a decimal representation of any rational number in Cantor's first argument specific list.

For example:

0 . 1 7 1 1 3 1 7 1 1 3 1 7 ...
1 . 0 0 0 0 0 0 0 0 0 0 0 0 ...
0 . 4 2 1 3 4 2 1 3 4 2 1 3 ...
0 . 1 0 1 0 1 0 1 0 1 0 1 0 ...
0 . 3 3 3 3 3 3 3 3 3 3 3 3 ...
2 . 0 0 0 0 0 0 0 0 0 0 0 0 ...
0 . 3 5 4 9 5 5 1 3 5 4 9 5 ...
3 . 0 0 0 0 0 0 0 0 0 0 0 0 ...
0 . 6 4 1 6 4 1 6 4 1 6 4 1 ...
0 . 3 0 2 0 3 0 2 0 3 0 2 0 ...
0 . 6 1 3 6 1 3 6 1 3 6 1 3 ...
0 . 2 7 1 0 2 7 1 0 2 7 1 0 ...
...
...
...

In this case Cantor's function result is 0.0101010101010101... which is not in the list.

Every non-zero decimal digit can be any number between 1 to 9, Because I use Cantor's function where the rules are:

A) Every 0 in the original diagonal number is turned to 1 in Cantor's new number.

B) Every non-zero in the original diagonal number is turned to 0 in Cantor's new number.

For example, there is no problem to use 0.333333... in the list because 3 (or any digit between 1 to 9) is always turned to 0, therefore our new number can always be a rational number.


Organic


Last edited by Organic on 10-21
 
Last edited:
  • #20
For example, there is no problem to use 0.333333... in the list because 3 (or any digit between 1 to 9) is always turned to 0, therefore our new number can always be a rational number.
You keep claiming "therefore our new number can always be a rational number" but you have given no proof of that. HOW can you prove that the new number WILL (not can) always be a rational number? Each digit of the new number is dependent upon the corresponding digit of a DIFFERENT rational number. HOW do you prove that those digits will either eventually be all zeroes or eventually repeat? That seems very unlikely to me.

For example, if the list of rational numbers between 0 and 1 starts (this is based on the standard listing used in the proof that rational numbers are countable):
0.50000...
0.33333...
0.66666...
0.25000...
0.50000...
0.75000...
0.40000...
0.16666...
0.60000...
etc.

then your function (set the digit to 1 if the original digit is 0, otherwise to 0) gives 0.000111101 etc. I see no evidence that this will eventually repeat. Since you are claiming that it will, it your responsibility to prove that.

By the way, please do not start getting "mystical" and appealing to different philosophies of mathematics. One's philosophy of mathematics has no bearing on how one proves theorems (with the possible exception of those weird "Brouwer-Constructionists!). I am certainly not a "Platonist" but I understood Hurkyl's statements completely.
 
  • #21
If you add .101010101010... ("10/99") to your list, then by changing the nth digit you are guaranteeing that the new diagonal number formed will NOT be .101010... no matter where you put it in the list.
What if your list contains, at some point:
A=.110110110...
B=.00110110110...
C=.111011101110...
D=.000111011101110...
...
where going from A to C you simply use an additional 1 in the pattern and from A to B, from C to D, from E to F etc. you append as many zeros to the FRONT OF THE NUMBER (not within the pattern) as there are leading 1's in the number to get the next number. These are all clearly rational.
doing a straight diagonal change on this one will give:
.010010001000010000010000001...
Which is irrational. Of course we could reorder things to try to avoid this. Also, if we just say arbitrarily change the digits on the diagonal to any other digits that will certainly complicate the argument.
Just some things to think about...
Aaron
 
  • #22
Well, I think it is depended on your point of view, which in this case is Platonism, which says that mathematics only discover timeless objective truth.

It does not depend on a point of view; it depends on the axiomatic definition of the quantifier "there exists".


My list is a decimal representation of any rational number in Cantor's first argument specific list.

Sorry, I meant to ask where 1/99 appears in the list.
 
  • #23
Hi Hurkyl,


.010101010... (1/99) is not in the list exactly as Cantor's function real number result is not in the list.

Therefore in both cases (R list or Q list) we have to add Cantor's function resutltes to the list.

Therefore the list is not complete and we can come to the conclusion that |N|<|R| or |N|<|Q|.


Organic.
 
Last edited:
  • #24
Hi Hurkyl,


If my original number is for example : 0.101010301010... then by using the rule that changes any 0 to 1 and any non-zero to 0 I define the new number 0.01010101010... which is not in the list.

0.101010301010... is not a rational number because it has not the exact repetition's pattern over scales, therefore it is not in the list.

So, in this case 0.01010101010... is the Cantor's function results which is not in the list and have to be added to the list.

This is the exact state between Cnator's function some result and the R list.

And I have found that this sate holds for Q list.

Therefore we can come to the conclusion that Cantor's second argument |N|<|R| but also |N|<|Q|.


Organic
 
  • #25
Hi synergy, Hi HallsofIvy,

Fact number 1: We can represent all rational numbers in their decimal form.

Fact number 2: Any rational number, which is represented in its decimal form, has aleph0 digits.

Therefore there are no limits to the number of ways that Q list can be rearranged to give us some new Q number (which is not in the list) as a Cantor's function result.

Therefore |N|<|Q| by Cantor's second argument.


Organic
 
Last edited:
  • #26
.010101010... (1/99) is not in the list exactly as Cantor's function real number result is not in the list.

Then why are you claiming:

My list is a decimal representation of any rational number in Cantor's first argument specific list.

When there is clearly at least one rational number whose decimal representation is not in your list?


This is the exact state between Cnator's function some result and the R list.

But there's a very important logical piece you're missing. Cantor's second diagonal argument is applied to EVERY list of real numbers, and that's how you conclude |N|<|R|.

However, you are applying the diagonal argument to a single list, and if it is the case that no rational number appears twice on the list, all we can conclude is |N|<=|Q|.


That is the key you are missing. In order to prove |S|<|T| for sets S and T, you have to prove that EVERY FUNCTION from S to T is missing an element of t. Proving it for only one function doesn't cut it1.


1: except when there exists only one function from S to T, in which case proving it for one function coincides with proving it for every function. This situation can only happen when S or T is the empty set, or when |S|=|T|=1.


Therefore there are no limits to the number of ways that Q list can be rearranged to give us some new Q number (which is not in the list) as a Cantor's function result.

Sure there is. There are at most |N|^|N| = c ways.

And I bet I can give you a list of rational numbers for which "Cantor's function" does not give you a rational number for any rearrangement. :smile:
 
Last edited:
  • #27
Hi Hyrkyl,

Sure there is. There are at most |N|^|N| = c ways.

And I bet I can give you a list of rational numbers for which "Cantor's function" does not give you a rational number for any rearrangement.

Ok, I'm waiting.



Thank you Hurkyl, my argument is wrong because I have at least one rational decimal form which is not in Cantor's diagonal first list.

Therefore, we can conclude that at least one rearrangement of all rational numbers decimal forms, can't give us a rational number as a result.

This is a beautiful insight for me,:smile: and you are a great mentor.


Organic
 
Last edited:
  • #28
Can Hurkyl be as vague in defining his list as you are in yours?
 
  • #29
Thank you Hurkyl, my argument is wrong because I have at least one rational decimal form which is not in my list, but exists in Cantor's diagonal first list.

Therefore, we can conclude that at least one rearrangement of all rational numbers decimal forms, can't give us a rational number as a result.

This is a beautiful insight for me,:smile: and you are a great mentor.


Organic
 
  • #30
So, I have a question.

Cantor's first diagonal argument is like the Rosetta stone (http://www.rosetta.com/RosettaStone.html [Broken]) for the decimal forms of rational numbers.

Do Irrational numbers can have some kind of "Rosetta Stone" ?


Organic
 
Last edited by a moderator:
  • #31
When we have a complete list of rational numbers, represented by their decimal form, then Cantor's function result cannot be but an irrational number.

I think we have here some interesting state, because if what I wrote holds, it means that there can be some difference between aleph0 and aleph0-1, which is not quantitative but structural.

It means that if even one of the rational numbers is missing, we have the ability to define some rational number (repetitions over scales) as Cantor's function result.

But when we have a complete list of rational numbers, represented by their decimal forms, then Cantor's function result cannot be but an irrational number (no repetitions over scales).

Is there some mathematical area which deals with this fine difference between aleph0 and aleph0-1 ?
 
Last edited:
  • #32
If we take the next step we can ask what is the structural difference between 2^aleph0 and 2^aleph0-1 ?

We sow that Cantor's function can find some rational number as a result (repetitions over scales), only if our list is aleph0-1.

Because we have only one representation to all irrational numbers, which is the base value expansion, how can we be sure that we are not in the same situation, which has been found in the case of the rational numbers list ?

In the case of the rational numbers we have two representation forms of numbers that can be compared to each other and help us to find the complete list.

But this is not the case of all R numbers that can be represented by only one form.

What I mean is when we still find some R number which is not in the list, can't we say that it means that we have 2^aleph0-1 list, which is not the 2^aleph0 complete list?

In this case we call |R| the uncountable, but maybe the only reason is the fact that we don’t know how to represent 2^aleph0 numbers, and all we have is the representation of 2^aleph0-1 numbers.

I know that 2^aleph0-1 = 2^aleph0, but again I talking about the structural difference between |R| and |R|-1.
 
Last edited:
  • #33
In this case we call |R| the uncountable, but maybe the only reason is the fact that we don’t know how to represent 2^aleph0 numbers, and all we have is the representation of 2^aleph0-1 numbers.
No, we call R "uncountable" because it fits the precise mathematical definition of the word "uncountable".

I know that 2^aleph0-1 = 2^aleph0, but again I talking about the structural difference between |R| and |R|-1.
Then perhaps it would be a good idea for you to tell us exactly what YOU mean by "|R|-1", as well as what you mean by the "structure" of a cardinality.
 
  • #34
Hi HallsofIvy,

You wrote:
...it would be a good idea for you to tell us...
How is us ? Are you more then a one person ?

And if not, it looks like you think that you are the speaker of some group.

In this case what is the name of the group that gave you the job to be their speaker ?

My advice to you is: Please be more modest and less aggressive to the persons who write their point of views on any subject in this forum. You can learn it from Hurkyl.

First of all Math is a form of language and any language is a communication tool between persons, even if its gremial is rigorous. Please don't forget this important point.

Some persons are professionals and some are not, but you never know where some good idea can appear.

If you understand what I wrote about the structural difference between aleph0 and aleph0-1, then take this understanding and connect it to 2^aleph0 and 2^aleph0-1.

When the list has aleph0-1 numbers, then Cantor's function result can be some rational number (repetitions over scales).

When the list has aleph0 numbers, then Cantor's function result can't be but some irrational number (no repetitions over scales).

These are structural differences.

When the list has 2^aleph0-1 numbers, then Cantor's function result is some R number (repetitions, or no repetitions over scales).

When the list has 2^aleph0 numbers, then Cantor's function result is unknown.




Organic
 
Last edited:
  • #35
Look up the phrase "editorial we". Yes, I was speaking on behalf of all the people (one or two at least) who read this thread. I certainly don't expect you to correspond with me personally to define your terms. I have no doubt that anyone reading this thread would appreciate you actually defining your terms.

You are completely correct that Hurkyl is "more modest and less aggressive" than I am. He perhaps still has some hopes that you will actually understand what he is saying.

I might also point out that you are not a good one to talk about being "more modest and less aggressive". You have repeatedly asserted that you knew for a fact that something that all professional mathematicians understand and accept is incorrect. You then give a lot of undefined terms and hand-waving to support that contention. Your basic argument appears to be "I do not understand this, therefore it is wrong."
 
<h2>1. What is Cantor's second diagonalization argument?</h2><p>Cantor's second diagonalization argument is a proof used in set theory to show that the cardinality (size) of the set of real numbers is greater than the cardinality of the set of natural numbers. It is also known as the "diagonal argument" or "Cantor's diagonal argument."</p><h2>2. How does Cantor's second diagonalization argument work?</h2><p>The argument involves constructing a list of all possible real numbers between 0 and 1, and then creating a new number by changing the digits along the diagonal of the list. This new number is not included in the list, which proves that the set of real numbers is uncountable (has a higher cardinality) compared to the set of natural numbers.</p><h2>3. What is the significance of Cantor's second diagonalization argument?</h2><p>Cantor's second diagonalization argument is significant because it proved that there are different sizes of infinity. This concept challenged the previously accepted idea that all infinite sets were the same size. It also had a major impact on the development of mathematics and set theory.</p><h2>4. Are there any criticisms of Cantor's second diagonalization argument?</h2><p>Yes, there have been some criticisms of Cantor's second diagonalization argument. Some argue that it relies on the assumption that the set of real numbers is uncountable, which has been debated by mathematicians. Others argue that the argument does not fully address the concept of infinity and that there may be other ways to conceptualize the size of infinite sets.</p><h2>5. How is Cantor's second diagonalization argument related to the concept of "uncountable" sets?</h2><p>Cantor's second diagonalization argument is often used to prove that a set is uncountable, meaning it has a higher cardinality than the set of natural numbers. This is because the argument shows that there are elements in the set that cannot be counted or listed in a one-to-one correspondence with the natural numbers, which is a key characteristic of uncountable sets.</p>

1. What is Cantor's second diagonalization argument?

Cantor's second diagonalization argument is a proof used in set theory to show that the cardinality (size) of the set of real numbers is greater than the cardinality of the set of natural numbers. It is also known as the "diagonal argument" or "Cantor's diagonal argument."

2. How does Cantor's second diagonalization argument work?

The argument involves constructing a list of all possible real numbers between 0 and 1, and then creating a new number by changing the digits along the diagonal of the list. This new number is not included in the list, which proves that the set of real numbers is uncountable (has a higher cardinality) compared to the set of natural numbers.

3. What is the significance of Cantor's second diagonalization argument?

Cantor's second diagonalization argument is significant because it proved that there are different sizes of infinity. This concept challenged the previously accepted idea that all infinite sets were the same size. It also had a major impact on the development of mathematics and set theory.

4. Are there any criticisms of Cantor's second diagonalization argument?

Yes, there have been some criticisms of Cantor's second diagonalization argument. Some argue that it relies on the assumption that the set of real numbers is uncountable, which has been debated by mathematicians. Others argue that the argument does not fully address the concept of infinity and that there may be other ways to conceptualize the size of infinite sets.

5. How is Cantor's second diagonalization argument related to the concept of "uncountable" sets?

Cantor's second diagonalization argument is often used to prove that a set is uncountable, meaning it has a higher cardinality than the set of natural numbers. This is because the argument shows that there are elements in the set that cannot be counted or listed in a one-to-one correspondence with the natural numbers, which is a key characteristic of uncountable sets.

Similar threads

Replies
11
Views
367
  • General Math
Replies
4
Views
698
  • Set Theory, Logic, Probability, Statistics
Replies
6
Views
1K
Replies
3
Views
379
  • General Math
Replies
24
Views
2K
  • Set Theory, Logic, Probability, Statistics
2
Replies
43
Views
3K
Replies
5
Views
803
  • General Math
Replies
2
Views
871
Replies
5
Views
1K
Replies
4
Views
825
Back
Top